AAMC MCAT Practice Exam 2

Your page rank:

Total word count: 18646
Pages: 68

Calculate the Price

- -
275 words
Looking for Expert Opinion?
Let us have a look at your work and suggest how to improve it!
Get a Consultant

C/P: What expression gives the amount of light energy (in J per photon) that is converted to other forms between the fluorescence excitation and emission events?

"intensity of fluorescence emission at 440 nm excitation at 360 nm) was monitored for 20 minutes"

A) (6.62 × 10-34) × (3.0 × 108)
B) (6.62 × 10-34) × (3.0 × 108) × (360 × 10-9)
C) (6.62 × 10-34) × (3.0 × 108) × [1 / (360 × 10-9) – 1 / (440 × 10-9)]
D) (6.62 × 10-34) × (3.0 × 108) / (440 × 10-9)

C) (6.62 × 10-34) × (3.0 × 108) × [1 / (360 × 10-9) – 1 / (440 × 10-9)] The answer to this question is C because the equation of interest is E = hf = hc/λ, where h = 6.62 × 10 −34 J ∙ s and c = 3 × 10 8 m/s. Excitation occurs at λe = 360 nm, but fluorescence is observed at λf = 440 nm. This implies that an energy of E = (6.62 × 10 −34) × (3 × 10 8) × [1 / (360 × 10 −9) − 1 / (440 × 10 −9)] J per photon is converted to other forms between the excitation and fluorescence events.

C/P: Compared to the concentration of the proteasome, the concentration of the substrate is larger by what factor?

"purified rabbit proteasome (2 nM) was incubated in the presence of porphyrin…the reaction was initiated by addition of the peptide (100 uM)"

A) 5 × 101
B) 5 × 102
C) 5 × 103
D) 5 × 104

D) 5 × 104 The answer to this question is D. The proteasome was present at a concentration of 2 × 10-9 M, while the substrate was present at 100 × 10-6 M. The ratio of these two numbers is 5 × 104.

sp2 hybridized

possess exactly one doubly bonded atom

C/P: The concentration of enzyme for each experiment was 5.0 μM. What is kcat for the reaction at pH 4.5 with NO chloride added when Compound 3 is the substrate?

Rate of reaction = 125 nM/s

A) 2.5 × 10-2 s-1
B) 1.3 × 102 s-1
C) 5.3 × 103 s-1
D) 7.0 × 105 s-1

A) 2.5 × 10-2 s-1 The answer to this question is A. The fact that the rate of product formation did not vary over time for the first 5 minutes implies that the enzyme was saturated with substrate. Under these conditions, kcat = Vmax/[E] = (125 nM/s)/5.0 μM = 2.5 × 10-2 s-1.

kcat, Vmax, [E]

kcat = Vmax/[E]

C/P: Absorption of ultraviolet light by organic molecules always results in what process?
A) Bond breaking
B) Excitation of bound electrons
C) Vibration of atoms in polar bonds
D) Ejection of bound electrons

B) Excitation of bound electrons The answer to this question is B. The absorption of ultraviolet light by organic molecules always results in electronic excitation. Bond breaking can subsequently result, as can ionization or bond vibration, but none of these processes are guaranteed to result from the absorption of ultraviolet light.

C/P: Four organic compounds: 2-butanone, n-pentane, propanoic acid, and n-butanol, present as a mixture, are separated by column chromatography using silica gel with benzene as the eluent. What is the expected order of elution of these four organic compounds from first to last?

A) n-Pentane → 2-butanone → n-butanol → propanoic acid
B) n-Pentane → n-butanol → 2-butanone → propanoic acid
C) Propanoic acid → n-butanol → 2-butanone → n-pentane
D) Propanoic acid → 2-butanone → n-butanol → n-pentane

A) n-Pentane → 2-butanone → n-butanol → propanoic acid The answer to this question is A. The four compounds have comparable molecular weights, so the order of elution will depend on the polarity of the molecule. Since silica gel serves as the stationary phase for the experiment, increasing the polarity of the eluting molecule will increase its affinity for the stationary phase and increase the elution time (decreased Rf).

C/P: The half-life of a radioactive material is:

A) half the time it takes for all of the radioactive nuclei to decay into radioactive nuclei.
B) half the time it takes for all of the radioactive nuclei to decay into their daughter nuclei.
C) the time it takes for half of all the radioactive nuclei to decay into radioactive nuclei.
D) the time it takes for half of all the radioactive nuclei to decay into their daughter nuclei.

D) the time it takes for half of all the radioactive nuclei to decay into their daughter nuclei. The answer to this question is D because the half-life of a radioactive material is defined as the time it takes for half of all the radioactive nuclei to decay into their daughter nuclei, which may or may not also be radioactive.

C/P: A person is sitting in a chair. Why must the person either lean forward or slide their feet under the chair in order to stand up?

A) to increase the force required to stand up
B) to use the friction with the ground
C) to reduce the energy required to stand up
D) to keep the body in equilibrium while rising

D) to keep the body in equilibrium while rising The answer to this question is D because as the person is attempting to stand, the only support comes from the feet on the ground. The person is in equilibrium only when the center of mass is directly above their feet. Otherwise, if the person did not lean forward or slide the feet under the chair, the person would fall backward due to the large torque created by the combination of the weight of the body (applied at the person’s center of mass) and the distance along the horizontal between the center of mass and the support point.

C/P: The side chain of tryptophan will give rise to the largest CD signal in the near UV region when:
A) present as a free amino acid
B) part of an a-helix
C) part of a B-sheet
D) part of a fully folded protein

D) part of a fully folded protein The answer to this question is D because tryptophan has an aromatic side chain that will give rise to a significant CD signal in the near UV region if it is found in a fully folded protein.

C/P: Which amino acid will contribute to the CD signal in the far UV region, but NOT the near UV region, when part of a fully folded protein?

"Asymmetry resulting from tertiary structural features causes the largest increase in CD signal intensity in the near UV region of peptides. The side chains of amino acid residues absorb in this region.

The peptide bond absorbs in the far UV region (190-250 nm). The CD signals of these bonds are dramatically impacted by their proximity to secondary structural elements."

A) Trp
B) Phe
C) Ala
D) Tyr

C) Ala

C/P: Based on the relative energy of the absorbed electromagnetic radiation, which absorber, a peptide bond or an aromatic side chain, exhibits an electronic excited state that is closer in energy to the ground state?

"Asymmetry resulting from tertiary structural features causes the largest increase in CD signal intensity in the near UV region of peptides. The side chains of amino acid residues absorb in this region.

The peptide bond absorbs in the far UV region (190-250 nm). The CD signals of these bonds are dramatically impacted by their proximity to secondary structural elements."

A) An aromatic side chain; the absorbed photon energy is higher.
B) An aromatic side chain; the absorbed photon energy is lower.
C) A peptide bond; the absorbed photon energy is higher.
D) A peptide bond; the absorbed photon energy is lower.

B) An aromatic side chain; the absorbed photon energy is lower. The answer to this question is B because aromatic side chains absorb in the near UV region of the electromagnetic spectrum, which has longer wavelengths, and hence lower energy, than peptide bonds. Because the energy of the photon matches the energy gap between the ground and the excited state, this implies that the aromatic side chain has more closely spaced energy levels.

C/P: What is the net charge of sT-loop at pH 7.2?

"A synthetic peptide with the amino acid sequence KTFCGPEYLA was generated as a mimic of the T-loop. This synthetic T-loop (sT-loop) was incubated with 32P-labeled ATP in the presence of PDK1 for different time periods at 37 ° C and pH 7.2, and the amount of radioactivity incorporated into sT-loop was measured by detection of β- decay."

A) -2
B) -1
C) 0
D) +1

C) 0 The answer to this question is C because at pH 7.2, the N-terminus will be positively charged and the C-terminus will be negatively charged. In addition, the lysine side chain will carry one positive charge and the glutamic acid side chain will carry one negative charge.

C/P: In designing the experiment, the researchers used which type of P-32 labeled ATP?
A) aP32-ATP
B) BP32-ATP
C) γP32-ATP
D) δP-32 ATP

D) δP-32 ATP The answer to this question is C because the phosphoryl transfer from kinases comes from the γ-phosphate of ATP. Therefore, the experiment should require γ32P-ATP.

C/P: When used in place of spHM, which peptide would be most likely to achieve the same experimental results?

"This experiment was repeated in the presence of a synthetic peptide that mimics the HM domain (sHM) of Ser/Thr kinases with the amino acid sequence FLGFTY. Phosphorylated sHM (spHM) was also used in place of sHM."

A) FLGFAY
B) FLGFQY
C) FLGFGY
D) FLGFEY

D) FLGFEY The answer to this question is D because the phosphorylated threonine would most likely be mimicked by glutamic acid in terms of size and charge.

C/P: Based on the information in the passage, PDK1 catalyzes the addition of phosphate to what functional group?

"This experiment was repeated in the presence of a synthetic peptide that mimics the HM domain (sHM) of Ser/Thr kinases with the amino acid sequence FLGFTY. Phosphorylated sHM (spHM) was also used in place of sHM."

A) Hydroxyl
B) Amine
C) Carboxyl
D) Phenyl

A) Hydroxyl The answer to this question is A because reactions involving either Ser or Thr would involve the hydroxyl group in the side chain of these amino acids.

C/P: Which statement about the cooperativity of RIα/C activation and RIα protein folding is supported by the data in figures 2 and 3?
A) Both activation and folding are cooperative.
B) Activation is cooperative, but folding is not.
C) Folding is cooperative, but activation is not.
D) Neither activation nor folding is cooperative

A) Both activation and folding are cooperative. The answer to this question is A because both curves have a sigmoidal shape, which is indicative of cooperative processes.

C/P: A patient puts on a mask with lateral openings and inhales oxygen from a tank.

Which phenomenon causes static air to be drawn into the mask when oxygen flows?

A) Doppler effect
B) Venturi effect
C) Diffusion
D) Dispersion

B) Venturi effect The answer to this question is B because oxygen pressure is the sum of the oxygen static pressure P and the oxygen flow pressure rv2/2. In the area of the mask openings, Pair = P + rv2/2, thus Pair > P. Air enters the mask because the static pressure of the air is larger than the static pressure of the oxygen in flow. This is the Venturi effect, and the mask is called the Venturi mask.

Doppler effect

an increase (or decrease) in the frequency of sound, light, or other waves as the source and observer move toward (or away from) each other This effect causes the sudden change noticeable in a passing siren, as well as the redshift seen by astronomers.

Venturi effect

reduction in fluid pressure that results when a fluid flows through a constricted section of a pipe

diffusion

net movement of molecules from a region of higher concentration to a region of lower concentration

dispersion

the separation of light into colors by refraction or diffraction with formation of a spectrum

C/P: What causes duplex DNA with a certain (A + T):(G + C) ratio to melt at a higher temperature than comparable length duplex DNA with a greater (A + T):(G + C) ratio?
A) Stronger van der Waals forces of pyrimidines
B) Stronger van der Waals forces of purines
C) Increased number of hydrogen bonds
D) Reduced electrostatic repulsion of phosphates

C) Increased number of hydrogen bonds The answer to this question is C. GC base pairs involve three hydrogen bonds, while AT base pairs involve only two. This disparity has often been used to explain the increased melting temperature of DNA rich in GC content.

C/P: Which property of a substance is best used to estimate its relative vapor pressure?
A) Melting point
B) Boiling point
C) Molecular weight
D) Dipole moment

B) Boiling point The answer to this question is B because of the properties listed, the boiling point of a substance will give the best estimate of its relative vapor pressure.

C/P: What are the structural features possessed by storage lipids?

A) Two fatty acids ester-linked to a single glycerol plus a charged head group
B) Three fatty acids ester-linked to a single glycerol
C) Two fatty acids ester-linked to a single sphingosine plus a charged head group
D) Three fatty acids ester-linked to a single sphingosine

B) Three fatty acids ester-linked to a single glycerol The answer to this question is B because triacylglycerols are neutral storage lipids. They consist of three fatty acids ester-linked to a single glycerol.

C/P: In the overall electrochemical reaction:

N2(g) + H2(g) –> NH3(g)

Half reactions: H2(g) → 2H+ + 2e-
N2(g) + 6H+ + 6e- → 2NH3(g)

A) nitrogen is oxidized at the anode, and hydrogen is reduced at the cathode.
B) nitrogen is reduced at the cathode, and hydrogen is oxidized at the anode.
C) nitrogen is reduced at the anode, and hydrogen is oxidized at the cathode.
D) nitrogen is oxidized at the cathode, and hydrogen is reduced at the anode.

B) nitrogen is reduced at the cathode, and hydrogen is oxidized at the anode. The answer to this question is B because oxidation always occurs at the anode and reduction at the cathode of an electrochemical cell. Since nitrogen decreases in oxidation state during the reaction, it is reduced. Hydrogen, on the other hand, increases in oxidation state and is, therefore, oxidized.

ANOX REDCAT

In an electrochemical cell: oxidation occurs at the anode reduction occurs at the cathode

C/P: In industrial use, ammonia is continuously removed from the reaction mixture. This serves to drive Reaction 1 because of:

N2(g) + H2(g) –> NH3(g)

A) Boyle’s law
B) Charles’s law
C) Heisenberg’s principle
D) Le Châtelier’s principle

D) Le Châtelier’s principle The answer to this question is D because removing a product as it forms causes a displacement from the equilibrium condition. The system will respond by shifting more reactants to the product side. This is an example of Le Châtelier’s principle.

Boyle’s law

a law stating that the pressure of a given mass of an ideal gas is inversely proportional to its volume at a constant temperature

Charles’ Law

a law stating that the volume of an ideal gas at constant pressure is directly proportional to the absolute temperature

Heisenberg principle

the position and the velocity of an object cannot both be measured exactly, at the same time, even in theory

Le Châtelier’s principle

a principle stating that if a constraint (such as a change in pressure, temperature, or concentration of a reactant) is applied to a system in equilibrium, the equilibrium will shift so as to tend to counteract the effect of the constraint

C/P: The lone pair of electrons in ammonia allows the molecule to:
A) assume a planar structure
B) acts as an oxidizing agent
C) act as a Lewis acid in water
D) act as a Lewis base in water

D) act as a Lewis base in water The answer to this question is D because, by definition, a Lewis base is a substance that donates an electron pair in forming a covalent interaction.

*C/P: What is the role of the solid-state catalyst in the Haber process?
A) It increases the amount of ammonia produced per unit time
B) It increases the total amount of ammonia produced
C) It decreases the amount of ammonia that decomposes per unit time.
D) It decreases the total amount of ammonia produced

A) It increases the amount of ammonia produced per unit time The answer to this question is A because catalysts increase the rate of chemical reaction, which is the amount of product formed per unit time.

C/P: It is possible to design a reactor where the SCY conductor and the nitrogen/ammonia electrode operate at different temperatures. Which combination of temperatures is expected to give the best results?

"SCY conductors are favored for this use because their proton conductivities increase substantially with temperature."

A) SCY temperature higher than electrode temperature
B) SCY temperature lower than electrode temperature
C) SCY temperature the same as electrode temperature
D) The temperature of the components does not make a difference.

A) SCY temperature higher than electrode temperature The answer to this question is A because the proton conductivity of SCY increases with increasing temperature, while the favorability of reaction decreases with overall temperature. It is therefore beneficial to maintain the SCY conductor at a higher temperature.

C/P: At 25°C, the formation of [Cu(NH3)4]2+ according to Equation 1 is most likely a:

"The value of the formation constant of [Cu(NH3)4]2+ is 5.6 x 1011 at 25°C."

A) spontaneous process with positive ΔG°.
B) spontaneous process with negative ΔG°.
C) nonspontaneous process with positive ΔG°.
D) nonspontaneous process with negative ΔG°.

B) spontaneous process with negative ΔG°. The answer to this question is B because the equilibrium constant for the reaction is very large (much greater than 1). This necessarily means that ΔG° is negative and the reaction is spontaneous.

C/P: In the stepwise formation of [Cu(NH3)4]2+ from [Cu(H2O)4]2+, which of the following ions would form in the second step?

"The formation of [Cu(NH3)4]2+ takes place in a stepwise manner with one water molecule being replaced by an ammonia molecule in each step."

A) [Cu(H2O)2(NH3)2]2+
B) [Cu(H2O)2(NH3)3]2+
C) [Cu(H2O)3(NH3)]2+
D) [Cu(H2O)3(NH3)2]2+

A) [Cu(H2O)2(NH3)2]2+ The answer to this question is A. The reaction was stated to proceed in four sequential steps, with each step involving the replacement of one water molecule by a molecule of ammonia. As a result, the product of the second step is [Cu(H2O)2(NH3)2]2+

C/P: Which of the following best describes the bonds between Cu2+ and the nitrogen atoms of the ammonia molecules in [Cu(NH3)4]2+?
A) Ionic
B) Covalent
C) Coordinate ionic
D) Coordinate covalent

D) Coordinate covalent The answer to this question is D because the Lewis acid-base interaction between a metal cation and an electron pair donor is known as a coordinate covalent bond.

ionic bond

complete transfer of valence electron(s) between atoms

covalent bond

chemical bond that involves the sharing of electron pairs between atoms

coordinate covalent bond

one atom donates its electrons to form the covalent bond without the other atom contributing

C/P: Consider the reaction shown in Equation 1 at equilibrium. Would the concentration of [Cu(NH3)4]2+ increase if the equilibrium were disturbed by adding hydrochloric acid?

[Cu(H2O)4]2+(aq) + 4NH3(aq) –> [Cu(NH3)4]2+(aq) + 4H2O(l)

A) Yes, because the equilibrium in Equation 1 would shift to the left
B) No, because the equilibrium in Equation 1 would shift to the left
C) Yes, because the equilibrium in Equation 1 would shift to the right
D) No, because the equilibrium in Equation 1 would shift to the right

B) No, because the equilibrium in Equation 1 would shift to the left The answer to this question is B. Hydrochloric acid will protonate ammonia in a Brönsted acid-base reaction and reduce the amount of ammonia present. The disturbed equilibrium responds in a way to restore ammonia, but this causes the amount of [Cu(H2O)2(NH3)2]2+ to decrease. This means that the equilibrium shifts to the left.

C/P: In [Cu(NH3)4]2+, the subscript 4 indicates which of the following?
A) The oxidation number of Cu only
B) The coordination number of Cu2+ only
C) Both the oxidation number of Cu and the coordination number of Cu2+ only
D) Neither the oxidation number of Cu nor the coordination number of Cu2+

B) The coordination number of Cu2+ only The answer to this question is B. Because ammonia is neutral, the number 4 reflects only the number of ammonia molecules that bind to the central Cu2+ cation and does not indicate anything about its oxidation number.

C/P: Why does NH3 displace H2O in the formation of [Cu(NH3)4]2+?

I. NH3 contains more lone pairs of electrons than H2O.
II. NH3 is a stronger Lewis base than H2O.
III. NH3 donates a lone pair of electrons more readily than does H2O.

A) I and II only
B) I and III only
C) II and III only
D) I, II, and III

C) II and III only The answer to this question is C because Reaction 1 is a Lewis acid-base reaction. The fact that the reaction proceeds in the forward direction indicates that NH3 is a better Lewis base toward Cu2+ than H2O. This also means that NH3 donates a lone pair of electrons more readily than does H2O.

C/P: Which of the following atoms will be expected to have the smallest second ionization energy?
A) Na
B) C
C) O
D) Ca

D) Ca The answer to this question is D. Metals have lower ionization energies than non-metals as long as the ionization event involves a valence electron. Since Na is an alkali metal, it has only one valence electron and has a large second ionization energy. Ca is an alkaline earth metal and has two valence electrons. It will therefore have the smallest second ionization energy of the four atoms listed, which include Na and two non-metals.

C/P: Which of the following species has the largest mass percent of oxygen?
A) H2O
B) CaCO3
C) CO2
D) HCO3-

A) H2O The answer to this question is A because mass percent of oxygen is calculated by multiplying the coefficient for oxygen in the formula of the substance by 16 and then dividing by the molar mass of the substance. Water has the highest percentage of oxygen by mass of the compounds listed (16/18) × 100 = 89%.

C/P: What is the pH of a buffer solution that is 0.2 M in HCO3- and 2 M in H2CO3? (Note: The first pKa of carbonic acid is 6.37.)
A) 4.37
B) 5.37
C) 6.37
D) 7.37

B) 5.37 The answer to this question is B. The pH of the solution can be calculated using the Henderson-Hasselbach equation: pH = pKa + log([base]/[acid]). Plugging in the values provided in the question gives pH = 6.37 + log(0.2/2) = 5.37.

Henderson-Hasselbalch equation

pH = pka + log [base]/[acid] pOH = pkb+ log [acid]/[base]

C/P: What is the concentration of Ca2+(aq) in a saturated solution of CaCO3? (Note: The solubility product constant Ksp for CaCO3 is 4.9 × 10-9.)

Ca2+(aq) + 2HCO3-(aq) –> CaCO3(s) + CO2(g) + H2O(l)

A) 2.4 × 10-4 M
B) 4.9 × 10-5 M
C) 7.0 × 10-5 M
D) 4.9 × 10-9 M

C) 7.0 × 10-5 M The answer to this question is C. The solubility product constant expression for CaCO3 is Ksp = [Ca2+][CO32-]. Since equal quantities of Ca2+(aq) and CO32-(aq) are produced when CaCO3 dissolves, this expression reduces to 4.9 × 10-9 = x2, or 49 × 10-10 = x2. This can be solved directly by taking the square root of each side.

C/P: An inflatable cuff was used to temporarily stop blood flow in an upper arm artery. While releasing the pressure to deflate the cuff, a stethoscope was used to listen to blood flow in the forearm. The blood pressure reading was 130/85. Given this information, which of the following statements is LEAST likely to be true?

A) 85 mmHg was the diastolic pressure.
B) Blood flow was heard when the pressure of the cuff was greater than 130 mmHg.
C) 130 mmHg was the systolic pressure.
D) Blood flow was heard when the pressure of the cuff was 90 mmHg.

B) Blood flow was heard when the pressure of the cuff was greater than 130 mmHg. The answer to this question is B because the question indicates that the cuff was inflated to temporarily stop blood flow in the artery. The systolic pressure is determined from the first sound of blood flow that can be heard once the pressure exerted by the inflatable cuff falls below the pressure in the artery. The blood pressure reading was 130/85, which indicates that blood flow started again when the pressure was 130 mmHg. Therefore, blood flow was not heard when the pressure of the cuff was greater than 130 mmHg. A is not the correct response because the lower number of the blood pressure reading represents the diastolic pressure. C is not the correct response because the higher number of the blood pressure reading represents the systolic pressure. D is not the correct response because blood flow would be heard when the pressure of the cuff is 90 mmHg, as this pressure is higher than the diastolic pressure.

C/P: Which statement correctly describes the structure of the DNA double helix?
A) Nitrogenous bases pair with other bases in the same purine or pyrimidine groups.
B) The two DNA strands of the double helix are oriented in the same direction.
C) The amount of guanine will equal the amount of cytosine in a DNA sequence.
D) Sugar-phosphate backbones form the interior of the double helix.

C) The amount of guanine will equal the amount of cytosine in a DNA sequence. The answer to this question is C because as guanine and cytosine form base pairs on opposite DNA strands, they will occur in equal amounts within a specific DNA sequence. A is incorrect because purines and pyrimidines do not form base pairs with other members within the same group. B is incorrect because the strands are antiparallel, meaning that they are oriented in opposite directions. D is incorrect because sugar-phosphate backbones form the exterior of the double helix.

C/P: A glass rod is rubbed with a silk scarf producing a charge of +3.2 × 10-9 C on the rod. (Recall that the magnitude of the proton and electron charges is 1.6 × 10-19 C.) The glass rod has:
A.5.1 × 1011 protons added to it.
B.5.1 × 1011 electrons removed from it.
C.2.0 × 1010 protons added to it.
D.2.0 × 1010 electrons removed from it.

D.2.0 × 1010 electrons removed from it. The answer to this question is D because the number of charges in excess can be computed as +3.2 × 10 -9 C/1.6 × 10 -19 C = +2.0 × 10 10. This means that the rod has an excess of positive charge, created by removing a number of +2.0 × 10 10 electrons from the material, as it is not possible to add protons in a manner described in this question.

C/P: Which single bond present in nitroglycerin is most likely the shortest?

"The average bond energies in kJ/mol of the C-H, C-O, C-C, and O-N single bonds present in nitroglycerin are 413, 358, 347, and 201, respectively."

A) C-H
B) C-O
C) C-C
D) O-N

A) C-H The answer to this question is A. All of the bonds listed are single bonds. Since hydrogen has a much smaller atomic radius than second period elements, the covalent bond between C and H is shorter than any of the other bonds listed.

C/P: Based on the passage, the magnitude of ΔH° (in kJ) for the decomposition of 2 moles of nitroglycerin at 25°C is closest to which of the following?
A) 500
B) 1000
C) 2000
D) 3000

D) 3000 The answer to this question is D. The value of ΔH° can be calculated using the data provided in Table 1 and applying Hess’s Law. Two moles of nitroglycerin produce 6 moles of CO2(g) and 5 moles of H2O(g). The value of ΔH° for this amount of nitroglycerin combusted is 2(364.0) – 6(393.5) – 5(241.8) = -2842 kJ/mol.

C/P: At STP, the volume of N2(g) produced by the complete decomposition of 1 mole of nitroglycerin would be closest to which of the following?

4C3H5N3O9(l) → 12CO2(g) + 10H2O(g) + 6N2(g) + O2(g)

A) 5 L
B) 10 L
C) 20 L
D) 30 L

D) 30 L The answer to this question is D. Based on the balanced equation provided, 4 moles of nitroglycerin produce 6 moles of N2(g). Therefore, 1 mole of nitroglycerin will produce 1.5 moles of N2(g). At STP 1.5 moles of N2(g) will occupy 33.6 L since the molar volume of an ideal gas at STP is 22.4 L/mol.

C/P Which single bond present in nitroglycerin is the LEAST polar?
A) C-H
B) C-O
C) C-C
D) O-N

C) C-C The answer to this question is C because bonds between identical atoms are likely to be the least polar. This is true in this case since each C atom is bonded to one nitro group only.

C/P: What is the average power consumed by a 64-year-old woman during the ascent of the 15-cm-high steps, if her mass is 54 kg?

Time to climb up 30 steps (s): 27

A) 10 W
B) 20 W
C) 40 W
D) 90 W

D) 90 W The answer to this question is D because the power consumed is P=PE /time = mgh/t. From Table 1, there are 30 steps and t = 27 s. Then P = (54 kg × 10 m/s2 × 30 steps × 0.15 m/step) / (27s) = 90 W.

C/P: How much work did an 83-year-old female do while stretching the rubber band to the limit of her strength?

Maximum stretching of an elastic band (cm): 20

The volunteers stretched an elastic band whose elastic constant is k = 200 N/m

A) 4 J
B) 5 J
C) 6 J
D) 7 J

A) 4 J The answer to this question is A because the work done is W = 0.5 × kx2 where x = 0.20 m. So W = 0.5 × 200 (N/m) × (0.2 m)2 = 4.0 J.

Spring potential energy

E = 1/2kx^2

C/P: What kind of image is formed by the lenses of the glasses worn by a 68-year-old male who sees an object 2 m away?

Focal length of glasses worn (cm): -50

A) Real and enlarged
B) Real and reduced
C) Virtual and enlarged
D) Virtual and reduced

D) Virtual and reduced The answer to this question is D because the lenses have a negative focal length which means they are diverging lenses. Such lenses form virtual and reduced images of objects situated at distances larger than the focal length.

C/P: What is the ratio of the minimum sound intensities heard by a 64-year-old male and a 74-year-old female?

64 year-old male minimum sound intensity: 20 dB

74 year-old female minimum sound intensity: 40 dB

A) 20
B) 40
C) 50
D) 100

D) 100 The answer to this question is D because the relative intensities of the two sound waves are 20 dB and 40 dB, respectively. The difference is 20 dB, meaning that the decimal log of the ratio of their intensities is 2, which means that the ratio of their intensities is 100.

C/P: What is the approximate percentage of a 10C sample left after the time it took a 75-year-old male to walk one lap around the gym? (Note: The half-life of 10C is 20 seconds.)

Time to walk five laps (s): 200

A) 5%
B) 10%
C) 25%
D) 75%

C) 25% The answer to this question is C because the 75-year-old person takes 200 s to walk 5 laps, which is on average 40 s per lap. This time represents two half-lives of 10C, so only 1/(22) = 0.25, or 25% of 10C is left.

C/P: Which of the following energy conversions best describes what takes place in a battery-powered resistive circuit when the current is flowing?
A) Electrical to thermal to chemical
B) Chemical to thermal to electric
C) Electric to chemical to thermal
D) Chemical to electric to thermal

D) Chemical to electric to thermal The answer to this question is D because the chemical energy of the battery elements is used as electrical energy to set the charge carriers in motion through the resistor, where they experience drag from the crystal lattice of the resistive conductor and dissipate their energy as heat from the resistor.

C/P: Protein secondary structure is characterized by the pattern of hydrogen bonds between:
A) backbone amide protons and carbonyl oxygens
B) backbone amide protons and side chain carbonyl oxygens
C) side chain hydroxyl groups and backbone carbonyl oxygens
D) side chain amide protons and backbone carbonyl oxygens

A) backbone amide protons and carbonyl oxygens The answer to this question is A because secondary structure is represented by repeated patterns of hydrogen bonds between the backbone amide protons and carbonyl oxygen atoms.

C/P: A 60-Ω resistor is connected in parallel with a 20-Ω resistor. What is the equivalent resistance of the combination?
A) 80 ohms
B) 40 ohms
C) 15 ohms
D) 3 ohms

C) 15 ohms The answer to this question is C because the equivalent resistance is given by the expression (1/(60) + 1/(20))-1 = 15.

standard conditions

25 degrees C (298 K) 1 atm 1 M concentrations

STP

standard temperature and pressure 0 degrees C (273) 1 atm

CARS: illusory correlation bias

phenomenon of perceiving a relationship between variables even when no such relationship exists A false association may be formed because rare or novel occurrences are more salient and therefore tend to capture one’s attention.

*CARS: Why does the author most likely mention "the role of chance" (paragraph 4)?

"The small sample size of clinical experiences and the low regard of physicians for the role of chance can lead to errors in estimates of the probability of benefit."

A) As part of an explanation of why clinical impressions may be flawed
B) As a response to concerns about the small sample size clinicians may use
C) As a feature that makes clinical experience superior to evidence-based medicine
D) As part of an argument against the use of intuition in medical practice

A) As part of an explanation of why clinical impressions may be flawed The answer to this question is A because paragraph 4 is the second paragraph of three paragraphs (paragraphs 3, 4, and 5) that develop the author’s claim in the preceding paragraph that "there are a number of reasons that clinical experience may provide insufficient or misleading data" (paragraph 2).

CARS: Why does the author most likely mention Norway?

"This approach does not, however, account for other aspects of patient care, such as patient preferences, economics, and ethical issues. Norway, for instance, is discouraging the use of the osteoporotic agent alendronate; although the drug decreases hip fractures, ninety high-risk women would have to be treated with the drug for three years to prevent one hip fracture at a cost that could bankrupt the country’s medical plan."

A) To provide an exemplar of clinical practice
B) To provide an exemplar of evidence-based medicine
C) To provide an example of accounting for economics
D)To provide an example of one drug being less effective than it seems

C) To provide an example of accounting for economics The answer to this question is C because after discussing the advantages of evidence-based medicine, the author writes of evidence based medicine: "This approach does not, however, account for other aspects of patient care, such as patient preferences, economics, and ethical issues. Norway, for instance, is discouraging the use of the osteoporotic agent alendronate; although the drug decreases hip fractures, ninety high-risk women would have to be treated with the drug for three years to prevent one hip fracture at a cost that could bankrupt the country’s medical plan" (final paragraph). The emphasis on the cost of a particular treatment is, of course, a reference to economic factors in health-care decision-making.

CARS: Suppose that rigorous scientific research demonstrated that physicians who use a clinically-based model provide better care and have better patient outcomes than those who use an evidence-based model. How would this finding affect passage claims?

A) It would undermine the views of those clinicians who see clinically-based experience as sacrosanct.
B) It would support the suggestion that the approach taken in Norway is very unlikely to be effective.
C) It would challenge the views of the economists and health experts, as represented in the passage.
D) It would support the idea that music is an apt metaphor for medical practice.

C) It would challenge the views of the economists and health experts, as represented in the passage. The answer to this question is C because economists and health experts are described in the first paragraph as minimizing the value of clinical judgment (viewing it as "uncontrollable, chaotic, and obeying few rules"), and the findings described in the question challenge this.

CARS: In Experiment 1, which of the following hypotheses would most reasonably account for the post-experimental statements made by those in the neutral condition?

A) performer’s anxiety need not adversely affect performance.
B) A performer’s anxiety can be reduced with no effect on performance.
C) An audience can affect a performance by reducing performance anxiety.
D) An audience can affect a performance by causing performance anxiety.

A) performer’s anxiety need not adversely affect performance. The answer to this question is A because those in the supportive-audience condition were "less likely than those in the neutral-audience condition to report feeling distracted by the observer or feeling stress while doing the task" (paragraph 4). This must mean that those in the neutral condition were relatively more likely to report distraction or stress, but the passage also tells us that they had better performance than those in the supportive condition. So, if the statement in option A is true and a performer can feel anxiety without a negative effect on performance, then this would explain the apparent discrepancy (between reported anxiety and performance) of those in the neutral condition.The answer to this question is A because those in the supportive-audience condition were "less likely than those in the neutral-audience condition to report feeling distracted by the observer or feeling stress while doing the task" (paragraph 4). This must mean that those in the neutral condition were relatively more likely to report distraction or stress, but the passage also tells us that they had better performance than those in the supportive condition. So, if the statement in option A is true and a performer can feel anxiety without a negative effect on performance, then this would explain the apparent discrepancy (between reported anxiety and performance) of those in the neutral condition.

CARS: A visitor to Skellig Michael who kissed its stone cross probably did so for which of the following reasons?

"Penitents from across Europe not only traveled to the rock but made the grueling 700-foot climb to the Needle’s Eye, its precipitous westernmost peak. The ordeal required the devout to crawl onto a horizontal slab of rock that projects with a dizzy precariousness from the summit and kiss a stone cross affixed to the end of the slab."

A) To fulfill a qualification for sainthood
B) To atone for wrongs committed
C) To be spared by Viking marauders
D) To be accepted into the monastery

B) To atone for wrongs committed B is the answer because the passage describes "penitents . . . [who] made the grueling . . . climb to the Needle’s Eye" (final paragraph), a climb which culminated in kissing the stone cross. Because the passage refers to "penitents," it is most likely that these people sought to kiss the cross in an act of atonement.

CARS: Given Plato’s claim that the spoken word is superior to the written word in the pursuit of truth, which of the following activities would he have been most likely to encourage today’s truth seekers to adopt?

A) Listening to candidates for political office debate each other on the radio
B) Participating in a group that discusses Phaedrus and other written works at a local university
C) Posing comments on Internet sites and using online chat rooms to discuss the issues of the day
D) Getting involved in an ethics discussion group hosted by a public library

D) Getting involved in an ethics discussion group hosted by a public library D is the answer, because Plato argues that "debating sharpened memories, corrected misconceptions, and produced truth" (paragraph 3). This option describes people actively participating in a public debate about ethics and would be the kind of activity that Plato would advise for those seeking truth.

CARS: Which of the following assertions, if true, would most support the author’s statement that "Egyptian culture dwindled under the Classical world’s onslaught" (paragraph 4)?

A) Greek scholars used discoveries by Egyptian astronomers to further their own studies.
B) Egyptian scribes invented a flowing script for use in keeping economic records.
C) Invaders from Alexander the Great to the Romans tried to integrate Egypt into their empires.
D) Renaissance scholars incorrectly identified Greek script as ideographic.

C) Invaders from Alexander the Great to the Romans tried to integrate Egypt into their empires. C is correct because it represents an attack (or "onslaught") on Egypt by the Classical world (including the Romans) and describes an effort to appropriate or incorporate Egypt and its culture into other empires, which would by definition diminish Egyptian culture.

CARS: Why does the passage author assert that sixteenth- or seventeenth-century readers would have "laughed" at a Hemingway short story?

I. Because they would have found the writing style quaint
II. Because the plots of short stories have changed radically since then
III. Because basic, essential details were missing

A) II only
B) III only
C) I and II only
D) I and III only

B) III only The answer is B because the passage explains that Hemingway treats his imagined or fictional audience as "insiders" who already know essential background details about the fiction, even though those details are not provided in the work. In the first paragraph, the author maintains that a sixteenth- or seventeenth-century audience (as opposed to Hemingway’s twentieth-century audience) would have "laughed" at Hemingway’s work because they "would have been totally unable to adapt to [the] demands" the work imposed on readers. Later in the passage, the author says that Hemingway’s omission of certain details "implies a degree of shared information that would likely have baffled a sixteenth-century reader." So option III is supported in the passage.

CARS: Why was the identity of the audience unproblematic for the oral storyteller who rode from town to town reciting tales?

A) The composition of the audience kept changing.
B) The plot of the tale was already familiar to the listeners.
C) The members of the audience were literally on the scene.
D) The novelty of the performance superseded any need for information.

C) The members of the audience were literally on the scene. The answer is C because the author explains that the fictional or imagined audience has been important "from the time when writing broke away from oral performance" (final paragraph). In an oral performance, of course, there were listeners present; the "fictional audience" comes into play when, as the author puts it, the "writer’s audience is not actually present during the writing process" (paragraph 2).

CARS: Based on the passage, of the following, which would be the best example of "mass movements initiated by one or a few magnets that would unexpectedly sweep across the entire world"?

A) One country’s revolution leads to unrest and revolution in other countries around the world.
B) A disease in one country spreads to other countries, causing a pandemic.
C) Technological advances in one country are marketed to other developed countries.
D) A strengthening of democratic principles in one country is commended by the leadership of many countries around the world.

A) One country’s revolution leads to unrest and revolution in other countries around the world. The answer is A because the passage constructs an analogy between the behavior of magnets and the circumstances in which historical events occur and become interesting. The author maintains that under such circumstances: "your society would . . . [witness] alliances shifting constantly and in a way that would be neither quite orderly nor quite random." He or she also describes the way "no movement would be a simple copy of what had just happened." Option A describes this kind of a process, in which a revolution in one country influences the history and politics of other countries, but these instances of "unrest and revolution" in other countries could never be carbon copies of the first revolution.

CARS: One can most reasonably infer from the passage discussion in the second paragraph that before 1920, there were NO:

A) economic studies that provided empirical data about expenditures.
B) methods available for accurately comparing the standards of living of different social groups.
C) studies designed to assess the economic effect of changes in industrial production.
D) nationwide studies of the expenditures of all social groups in an industrial economy.

D) nationwide studies of the expenditures of all social groups in an industrial economy.

B/B: The information in the passage suggests that in mice CRY1 most likely affects XPA by:

<b>Increased CRY1 causes XPA levels to decrease</b>

A) activating XPA protein activity.
B) activating translation of XPA-encoding transcripts.
C) repressing replication of the XPA-encoding gene.
D) repressing transcription of the XPA-encoding gene.

D) repressing transcription of the XPA-encoding gene. The answer to this question is D because Figure 1 shows that XPA levels decrease as CRY1 levels increase.

B/B: Which cells harvested from adult mice were most likely used as the highly proliferative benchmark in the experiment that generated the data shown in Figure 3?
A) Adipocytes
B) Cardiac muscle cells
C) Gastrointestinal epithelial cells
D) Neurons

C) Gastrointestinal epithelial cells The answer to this question is C because the epithelial cells that line the gastrointestinal tract are typically highly proliferative.

B/B: After a section of a DNA strand containing a UVR-induced lesion is removed and resynthesized, the newly synthesized strand is rejoined to the remainder of the DNA strand by what type of bond?
A) Disulfide
B) Hydrogen
C) Peptide
D) Phosphodiester

D) Phosphodiester he answer to this question is D because phosphodiester bonds link the 3ʹ carbon atom of one deoxyribose and the 5ʹ carbon atom of another deoxyribose within the DNA molecules.

Purines vs. Pyrimidines

Purines: A, G (2 rings) Pyrimidines: T, C (1 ring) G-C bond has 3 H bonds (higher melting temperature) A-T bond has 2 H bonds

B/B: AlP exposed to an aqueous solution in which pH range will result in the largest amount of phosphine production?

AIP + 3H+ –&gt; Al3+ + PH3

A) pH &lt; 4
B) 4 &lt; pH &lt; 7
C) 7 &lt; pH &lt; 10
D) pH &gt; 10

A) pH < 4 The answer to this question is A because H+ is a reactant, and the increase in the concentration of H+ at low pH will favor product formation.

B/B: Based on the passage, which amino acid will most likely react with phosphine?

"It is thought that phosphine reacts with important sulfhydryl groups in complexes I (NADH dehydrogenase), II (succinate dehydrogenase), and IV (cytochrome oxidase) of the electron transport chain (ETC)."

A) Met
B) Cys
C) Ser
D) Thr

B) Cys The answer to this question is B because the passage states that phosphine reacts with sulfhydryl groups, and the cysteine side chain contains a sulfhydryl group.

B/B: When researchers determined the total cellular concentration of ATP in AlP-exposed rat liver cells, they found the concentration to be equal to the control value. Which conclusion about the metabolic state of the cell is best supported by these data?

A) Glycolytic flux is increased after AlP treatment.
B) Glycolytic flux is decreased after AlP treatment.
C) Citric acid cycle flux is increased after AlP treatment.
D) Citric acid cycle flux is decreased after AlP treatment.

A) Glycolytic flux is increased after AlP treatment. The answer to this question is A because ATP production is the same in both control and AlP-exposed cells, and the data in the passage show that mitochondrial ATP production is decreased. This indicates that the flux through glycolysis is increased, because this would be the major pathway for ATP production once the electron transport chain is shut down.

B/B: Why was it necessary for the researchers to determine the activity of the complexes independent of one another?

A) Complex stability is lost if the complexes are able to interact structurally.
B) The complexes have different cellular locations, and it is not feasible to isolate them together.
C) The complexes all use the same substrates, so their use must be monitored separately.
D) The reactions catalyzed by the complexes are coupled to one another.

D) The reactions catalyzed by the complexes are coupled to one another. The answer to this question is D because studying the complexes all together would lead to erroneous results because inhibition of complexes I and II affects the activity of Complex III, which affects the activity of Complex IV.

B/B: A large carbohydrate is tagged with a fluorescent marker and placed in the extracellular environment around a macrophage. The macrophage ingests the carbohydrate via phagocytosis. Which cellular structure is most likely to be fluorescently labeled upon viewing with a light microscope soon after phagocytosis?
A) Nucleus
B) Golgi apparatus
C) Lysosome
D) Endoplasmic reticulum

C) Lysosome The answer to this question is C because when a macrophage ingests foreign material, the material initially becomes trapped in a phagosome. The phagosome then fuses with a lysosome to form a phagolysosome. Inside the phagolysosome, enzymes digest the foreign object. Of the cell structures listed, the labeled carbohydrate is most likely to be microscopically visualized within a lysosome (phagolysosome).

nucleus

a dense organelle present in most eukaryotic cells, typically a single rounded structure bounded by a double membrane, containing the genetic material

Golgi apparatus

a complex of vesicles and folded membranes within the cytoplasm of most eukaryotic cells, involved in secretion and intracellular transport

lysosome

organelle in the cytoplasm of eukaryotic cells containing degradative enzymes enclosed in a membrane

endoplasmic reticulum

network of membranous tubules within the cytoplasm of a eukaryotic cell, continuous with the nuclear membrane usually has ribosomes attached as in involved in protein and lipid synthesis

B/B: Inhibition of phosphofructokinase-1 by ATP is an example of:
I. allosteric regulation
II. feedback inhibition
III. competitive inhibition

A) I only
B) II only
C) I and II only
D) II and III only

C) I and II only The answer to this question is C because ATP, the end product of glycolysis, downregulates through feedback inhibition the activity of phosphofructokinase-1 by binding to a regulatory site other than the active site of the enzyme (allosteric regulation). In contrast, competitive inhibition involves competition for binding to the active site.

B/B: Which process moves chlorine ions into the cells of the green algae?

A) Osmosis
B) Diffusion
C) Active transport
D) Facilitated diffusion

C) active transport The answer to this question is C because in order to maintain a higher concentration of chlorine ions inside the cell, the ions must be moved into the cell against their concentration gradient, which requires energy. This process is active transport. In the other processes, ions would move along their concentration gradient, either with or without the help of transport proteins.

B/B: Dendrotoxin from the mamba snake blocks voltage-gated potassium channels in somatic motor neurons that regulate skeletal muscle contraction. In what way would initial exposure to dendrotoxin affect the ability of a somatic motor neuron to propagate an electrical signal in response to a stimulus?
A) It would inhibit the initiation of an action potential.
B) It would shorten the refractory period.
C) It would prolong the action potential
D) It would prevent depolarization

C) It would prolong the action potential The answer to this question is C because if potassium ion channels are blocked, the membrane would fail to repolarize, extending the length of the action potential and simulating excessive muscle contractions.

B/B: Based on the passage, which statement describes Wnt proteins?

"Wnt proteins are a family of palmitoylated secretory proteins with isoelectric points around 9 that bind and activate the G protein-coupled receptor Frizzled, whose structure includes seven transmembrane α-helical domains."

A) They are composed of multiple subunits.
B) They have a positive charge.
C) They are synthesized in the smooth endoplasmic reticulum.
D) They fold into their tertiary structure in the cytoplasm.

B) They have a positive charge. The answer to this question is B because based on the passage, Wnt proteins are a family of secretory proteins with isoelectric points around 9, implying that they are positively charged at physiological pH. A is incorrect because there is no information in the passage to support this response. C is incorrect because secretory proteins are synthesized in the rough endoplasmic reticulum. D is incorrect because folding of secretory proteins occurs in the rough endoplasmic reticulum.

Based on the passage, β-catenin most likely has:

"Additional proteins in this pathway include the enzyme casein kinase-1 (CK1), glycogen synthase kinase-3 (GSK3), and β-catenin, which activates expression of Wnt target genes."

A) multiple subunits
B) very few disulfide bonds
C) a nuclear localization sequence
D) a high proportion of surface-exposed nonpolar residues

C) a nuclear localization sequence The answer to this question is C because according to Figure 1, β-catenin activates transcription factors for Wnt target genes. As transcription factors are found in the nucleus, β-catenin must enter the nucleus. Proteins that are translocated into the nucleus usually contain a nuclear localization sequence.

B/B: In the absence of Frizzled activation, β-catenin is covalently modified and:

"In the absence of Frizzled activation, CK1 and GSK3 sequentially phosphorylate β-catenin, which targets it for ubiquitination."

A) bound by a proteasome to initiate degradation into short peptides.
B) translocated into the Golgi body for secretion through exocytosis.
C) engulfed by a lysosome where it is hydrolyzed by proteases.
D) stored in vesicles until the signaling pathway is activated.

A) bound by a proteasome to initiate degradation into short peptides. The answer to this question is A because according to the passage and Figure 1, in the absence of Frizzled activation, β-catenin is phosphorylated and ubiquitinated. Ubiquitination targets a protein for degradation by a proteasome.

B/B: The middle region, as opposed to either end, of each of the seven α-helical domains of Frizzled is most likely to contain a high proportion of which type of amino acid residue?

"Wnt proteins are a family of palmitoylated secretory proteins with isoelectric points around 9 that bind and activate the G protein-coupled receptor Frizzled, whose structure includes seven transmembrane α-helical domains."

A) Nonpolar
B) Polar uncharged
C) Positively charged
D) Negatively charged

A) Nonpolar The answer to this question is A because according to the passage, the seven α-helical domains are transmembrane domains and, thus, cross the phospholipid bilayer. As a result, these domains are most likely to have a high proportion of hydrophobic residues. Hydrophobic residues are nonpolar.

B/B: Subunits of Protein X are linked covalently by bonds between the:
A) thiol groups of methionine residues
B) thiol groups of cysteine residues
C) hydroxyl groups of serine residues
D) hydroxyl groups of threonine residues

B) thiol groups of cysteine residues The answer to this question is B because Figure 1 shows that reducing agents separate subunits of Protein X. This indicates that subunits of Protein X are linked together by disulfide bonds, which implicate the thiol groups of cysteine residues.

B/B: What event will most likely occur if Protein X is inserted into the inner membrane of mitochondria?
A) The citric acid cycle will cease to function.
B) The electron transport chain will cease to function.
C) The proton gradient across the inner membrane will dissipate.
D) The pH of the intermembrane space will decrease.

C) The proton gradient across the inner membrane will dissipate. The answer to this question is C because based on information from the passage, Protein X forms membrane-spanning channels that alter the permeability of the inner membrane, thereby dissipating the proton gradient across the inner membrane.

B/B: The enzyme encoded by the tdh2 gene (glyceraldehyde-3-phosphate dehydrogenase) catalyzes the reversible conversion of:

A) 3-phosphoglycerate to 1,3-bisphosphoglycerate.
B) glyceraldehyde-3-phosphate to 1,3-bisphosphoglycerate.
C) fructose-1,6-bisphosphate to glyceraldehyde-3-phosphate.
D) 2-phosphoglycerate to 3-phosphoglycerate.

B) glyceraldehyde-3-phosphate to 1,3-bisphosphoglycerate The answer to this question is B because GAPDH, the enzyme encoded by the tdh2 gene, is a glycolytic enzyme that catalyzes the reversible conversion of glyceraldehyde-3-phosphate to 1,3-bisphosphoglycerate.

B/B: What is the best experimental method to analyze the effect of tdh2 gene deletion on the rate of histone acetylation? Comparing histone acetylation in wild-type and Δtdh2 cells by:
A) Western blot
B) Southern blot
C) Northern blot
D) RT-PCR

A) Western blot The answer to this question is A because posttranslational modification of proteins such as histone acetylation is analyzed by Western blotting.

Western blot

common method to detect and analyze proteins

Southern blot

a procedure for identifying specific sequences of DNA, in which fragments separated on a gel are transferred directly to a second medium on which detection by hybridization may be carried out.

Northern blot

technique used in molecular biology research to study gene expression by detection of RNA in a sample

B/B: Which experimental approach(es) can be used to analyze the effect of ROS on the lifespan of yeast? Comparing the lifespans of:

I. wild-type yeast versus yeast lacking antioxidant enzymes
II. wild-type yeast versus yeast overexpressing antioxidant enzymes
III. yeasts growing in the presence or absence of hydrogen peroxide

A) I only
B) II only
C) II and III only
D) I, II, and III

D) I, II, and III The answer to this question is D because all listed options influence ROS levels in yeast and can be used to analyze the role of ROS in regulating the lifespan of yeast.

B/B: Vasopressin regulates the insertion of aquaporins into the apical membranes of the epithelial cells of which renal structure?
A) Collecting duct
B) Proximal convoluted tubule
C) Bowman’s capsule
D) Ascending loop of Henle

A) Collecting duct The answer to this question is A because vasopressin regulates the fusion of aquaporins with the apical membranes of the collecting duct epithelial cells.

B/B: What are the primary myelin-forming cells in the peripheral nervous system?
A) Microglia
B) Astrocytes
C) Schwann cells
D) Oligodendrocytes

C) Schwann cells The answer to this question is C because Schwann cells are the myelin-forming cells in the peripheral nervous system.

B/B: Osmotic pressure Π is given by the relation:

Π = iMRT

where i is the van’t Hoff factor, M is the concentration of solute, R is the gas constant, and T is the temperature. The osmotic pressure of sea water is approximately 24 atm at 25°C. What is the approximate concentration of salt in sea water (approximated by NaCl with i = 2)? (Note: Use R = 0.08 L•atm/mol•K.)

A) 0.25 M
B) 0.50 M
C) 0.75 M
D) 1.0 M

B) 0.50 M The answer to this question is B since algebraic manipulation of the relation gives M = Π/iRT = 24 atm / (2 × 0.08 L•atm/mol•K × 300 K) = 24 / (2 × 24) mol/L = 0.5 M.

B/B: A prion is best described as an infectious:
A) prokaryote
B) transposon
C) protein
D) virus

C) protein The answer to this question is C because a prion is an abnormally folded protein that induces a normally folded version of the protein to also adopt the abnormal structure, which is often deleterious.

B/B: Assume that S. typhi immediately enters the bloodstream from the small intestine. Of the following, which would be the first major organ that bloodborne S. typhi would encounter?
A) Stomach
B) Pancreas
C) Large intestine
D) Liver

D) Liver The answer to this question is D because blood from the small intestine is transported first to the liver, which regulates nutrient distribution and removes toxins from the blood.

B/B: Researchers have noted that chloramphenicol (a commonly used antibiotic) is becoming less effective in treating typhoid fever. The best explanation for this observation would be selection:

A) against chloramphenicol in ΔF508 heterozygotes.
B) against chloramphenicol in wild-type homozygotes.
C) for chloramphenicol resistance in populations of S. typhi.
D) for ΔF508 CFTR, which cannot bind chloramphenicol.

C) for chloramphenicol resistance in populations of S. typhi. The answer to this question is C because according to the passage, S. typhi is the causative agent of typhoid fever. As S. typhi becomes resistant to chloramphenicol, this antibiotic will become less effective in treating typhoid fever.

B/B: G542X is another CFTR allele. If a female heterozygous for G542X bears a child fathered by a male heterozygous for the ΔF508 allele, what is the probability that the child would be homozygous for the G542X allele, given that neither parent has CF?

"The CFTR gene is found on an autosome. The most common mutation of the CFTR protein (ΔF508) results in a deleted phenylalanine at amino acid residue 508. Individuals homozygous for this allele exhibit CF."

A) 0.00
B) 0.25
C) 0.75
D) 1.00

A) 0.00 The answer to this question is A because both parents would need to carry the G542X allele in order for a child to be homozygous for the G542X allele. Because only the mother carries the G542X allele, the probability that the child will be homozygous for the G542X allele is 0.

B/B: Which of the following best describes the phenotype of an individual who is heterozygous with one ΔF508 and one wild-type CFTR allele?

"The CFTR gene is found on an autosome. The most common mutation of the CFTR protein (ΔF508) results in a deleted phenylalanine at amino acid residue 508. Individuals homozygous for this allele exhibit CF.

One hypothesis is heterozygote superiority. Under this hypothesis, individuals heterozygous for ΔF508 would have increased resistance to typhoid fever."

A) More susceptible to typhoid fever than wild-type homozygotes and has CF
B) More susceptible to typhoid fever than ΔF508 homozygotes and does not have CF
C) More resistant to typhoid fever than ΔF508 homozygotes and has CF
D) More resistant to typhoid fever than wild-type homozygotes and does not have CF

D) More resistant to typhoid fever than wild-type homozygotes and does not have CF The answer to this question is D because the passage states that individuals homozygous for the ΔF508 allele have CF. Therefore, an individual who is heterozygous for the ΔF508 allele does not have CF. However, this individual is more resistant to typhoid fever because the passage states that human epithelial cells expressing ΔF508 ingest significantly fewer S. typhi than do epithelial cells that express wild-type CFTR.

B/B: NPY is not classified as a releasing factor because it has:

"These factors are known as neuromodulators, in contrast to releasing factors, because they do not stimulate LH release on their own, but modulate (enhance or inhibit) the responses of the secretory cells of the anterior pituitary to GnRH."

A) no effect on LH secretion.
B) no effect on LH secretion without GnRH.
C) no effect on LH secretion in combination with GnRH.
D) the same effect on LH secretion as GnRH.

B) no effect on LH secretion without GnRH. The answer to the question is B because LH levels in the control and NPY alone treatment conditions were similar, and GnRH had to be present for LH levels to be significantly different from control levels. According to the passage, releasing factors stimulate LH release on their own. Therefore, NPY cannot be classified as a releasing factor because it had no effect on LH secretion without GnRH. In this case, NPY functioned as a neuromodulator.

B/B: Scientists are hopeful that NPY can be used in combination with GnRH to treat certain cases of female infertility. Individuals with a deficiency in what receptor system would be most likely to benefit from such a treatment?

A) GnRH
B) LH
C) NPY
D) LH and NPY

A) GnRH The answer to this question is A because according to the data in Table 1, NPY enhances the normal function of GnRH in stimulating blood LH levels.

B/B: Scientists have hypothesized that NPY is necessary for the generation of the preovulatory LH surge, a hormonal event that triggers ovulation. Which of the following findings best supports this hypothesis?

A) When the actions of NPY are blocked in female rats, the LH surge and ovulation do not occur.
B) NPY release from the hypothalamus increases just prior to the preovulatory LH surge.
C) NPY can enhance GnRH-stimulated LH secretion in female rats during the preovulatory period.
D) NPY has no effect on GnRH-stimulated LH secretion in male rats.

A) When the actions of NPY are blocked in female rats, the LH surge and ovulation do not occur. The answer to this question is A because when the function of only NPY is blocked, the LH surge and ovulation no longer occur. This provides strong evidence that NPY function is required for these events. B is incorrect because NPY release could occur just prior to the preovulatory LH surge, but these could be completely unrelated events. C is incorrect because in this case, NPY is enhancing the LH secretion, but it does not prove that NPY is necessary for the generation of the preovulatory LH surge. D is incorrect because ovulation occurs in females.

B/B: NPY amplified pituitary responses to GnRH by:

GnRH alone: 4.74 ng/mL
GnRH + NPY: 7.03 ng/mL

A) 27%.
B) 48%.
C) 67%.
D) 83%.

B) 48%. The answer to this question is B. From Table 1, GnRH + NPY = 7.03 and GnRH alone = 4.74. 7.03 / 4.74 = 1.48, so NPY amplified pituitary responses to GnRH by 48%.

B/B: Based on the passage, CatB and CatL most likely act on EGP in which of the following cellular compartments to facilitate membrane fusion?
A) Endoplasmic reticulum
B) Golgi apparatus
C) Endosomes
D) Cytosol

C) Endosomes The answer to this question is C because based on the passage, the entry of the virus into the host cell requires CatB and CatL proteases and involves endocytosis through the fusion of the viral membrane with the host cell membrane. Internalization of viral particles through endocytosis is mediated by endosomes.

Cytosol

aqueous component of the cytoplasm of a cell, within which various organelles are suspended

Based on the passage, CatB or CatL or both would be expected to have which of the following effects, if any, on EGP?

"mammalian proteases CatB and CatL"

A) No effect
B) Reduction of enzyme activity
C) Formation of protein dimers
D) Digestion into smaller protein fragments

D) Digestion into smaller protein fragments The answer to this question is D because the passage states that CatB and CatL are proteases. Proteases function to digest proteins into smaller fragments.

B/B: The precursor of EGP is translated from a transcript that has had one nontemplated nucleotide added to the open reading frame. This change does not create or eliminate a stop codon. Compared with the protein sGP, which is produced from the unedited transcript, EGP most likely has the same primary:

A) amino-terminal sequence as sGP, but a different primary carboxy-terminal sequence.
B) carboxy-terminal sequence as sGP, but a different primary amino-terminal sequence.
C) sequence as sGP except that EGP has one additional amino acid.
D) sequence as sGP except that EGP has one less amino acid.

A) amino-terminal sequence as sGP, but a different primary carboxy-terminal sequence. he answer to this question is A because the addition of one nucleotide to the open reading frame of EGP results in a frameshift mutation and an aberrant carboxy-terminal domain.

B/B: What molecule is NOT formed during the citric acid cycle?
A) Malate
B) Succinate
C) a-Ketoglutarate
D) Phosphoenolpyruvate

D) Phosphoenolpyruvate The answer to this question is D because phosphoenolpyruvate is a product of glycolysis, not the citric acid cycle.

B/B: Under anaerobic conditions, how many molecules of ATP are produced by the consumption of 5 moles of glucose?
A) 3 × 1024
B) 6 × 1024
C) 9 × 1024
D) 1.2 × 1025

B) 6 × 1024 Under anaerobic conditions, 2 moles of ATP are produced from each mole of glucose. Thus, 10 moles of ATP would be generated from 5 moles of glucose. Since there are 6 × 1023 molecules per mole, 10 moles of ATP is equal to 6 × 1024 molecules.

B/B: In humans, eggs and sperm are most similar with respect to:
A) cell size
B) genome size
C) the time required for development
D) the numbers produced by a single individual

B) genome size The answer to this question is B because both eggs and sperm contain a haploid number of chromosomes and therefore they are most similar with respect to their genome size.

In humans, the characteristic tissue of which of the following organs is NOT derived from mesoderm?
A) Brain
B) Heart
C) Kidney
D) Skeletal muscle

A) Brain The answer to this question is A because the brain is part of the central nervous system, which is derived from ectoderm. Heart, kidney, and skeletal muscle are derived from mesoderm.

B/B: Based on the passage, myosin Va most likely directly binds:

"myosin Va, a motor protein on microfilaments"

A) tubulin
B) actin
C) melanin
D) Rab27a

B) actin The answer to this question is B because based on the passage, myosin Va is a motor protein. Motor proteins such as myosin Va move along microfilaments through interaction with actin.

B/B: Secretory lysosomes are classified as lysosomes because secretory lysosomes have some functional components in common with conventional lysosomes. Given this, secretory lysosomes most likely contain:
A) ribosomes
B) Krebs cycle enzymes
C) RNA and DNa polymerases
D) degradative functions that function at low pH

D) degradative functions that function at low pH The answer to this question is D because lysosomes are defined as membrane-bound organelles that contain hydrolytic enzymes activated by a low pH. These enzymes are capable of degrading many kinds of biomolecules.

B/B: Melanosomes most likely move along microtubules that originate in and radiate from the:
A) centrosome
B) kinetochores
C) Golgi apparatus
D) microfilaments under the plasma membrane

A) centrosome The answer to this question is A because microtubules are cellular structures that originate from centrosomes.

centrosome

an organelle near the nucleus of a cell which contains the centrioles (in animal cells) and from which the spindle fibers develop in cell division

kinetochore

a complex of proteins associated with the centromere of a chromosome during cell division, to which the microtubules of the spindle attach

B/B: Lytic granules are generally released from CTLs when the T-cell receptors on these cells bind specifically to:

A) viral antigens presented on the surface of virus-infected cells.
B) growth factors secreted by helper T lymphocytes.
C) B-cell receptors on activated B lymphocytes.
D) constant regions of secreted antibodies.

A) viral antigens presented on the surface of virus-infected cells. The answer to this question is A because cytotoxic T lymphocytes target virus-infected cells by recognizing the viral antigen presented on the cell surface.

Based on the passage, Foxp3 affects ErbB2 expression in noncancerous mammary epithelium most likely by directly:

"Foxp3 binds specific sequences in the ErbB2 promoter, and deletion of these Foxp3-binding sites increases expression from the ErbB2 promoter in Foxp3-expressing cells."

A) inhibiting synthesis of ErbB2 mRNA.
B) stimulating synthesis of ErbB2 mRNA.
C) inhibiting synthesis of ErbB2 protein.
D) stimulating synthesis of ErbB2 protein.

A) inhibiting synthesis of ErbB2 mRNA. The answer to this question is A because the passage states that cancerous mammary epithelium in female Foxp3sf/+ mice has 8- to 12-fold more ErbB2 mRNA than noncancerous mammary epithelium. Because the passage states that Foxp3 binds the ErbB2 promoter, Foxp3 regulation of ErbB2 expression occurs at the transcriptional, not translational level. Noncancerous epithelium has reduced ErbB2 mRNA expression compared with cancerous epithelium, which suggests that in noncancerous epithelium, Foxp3 inhibits synthesis of ErbB2 mRNA.

B/B: Given that the tumorigenicity of a certain mouse mammary tumor cell line is dependent on ErbB2-mediated intracellular signaling, mice injected with variants of this cell line that have which of the following modifications would be most likely to survive the longest?

A) The Foxp3 genes deleted
B) One of the ErbB2 genes amplified
C) A Foxp3-expressing plasmid introduced
D) The Foxp3 binding sites deleted in the promoter of one of the ErbB2 genes

C) A Foxp3-expressing plasmid introduced The answer to this question is C because according to the passage, cancerous tumors are associated with the inactivating scurfin allele, not the wild-type Foxp3 allele. Because Foxp3 regulates ErbB2 transcription and high levels of ErbB2 mRNA are found in cancerous epithelium, the best modification to a cell line would be Foxp3-expressing plasmid, to repress transcription of ErbB2.

B/B: In a female mouse born with two wild-type Foxp3 alleles, what is the minimum number of inactivating, recessive mutations that might be sufficient in the Foxp3 alleles in mammary epithelium to significantly increase the likelihood that this mouse will develop mammary tumors?
A) 1
B) 2
C) 3
D) 4

A) 1 The answer to this question is A because the passage indicates that approximately 60% of female Foxp3sf/+ mice, which carry one inactivating scurfin (sf) allele and one wild-type (+) allele, spontaneously develop mammary tumors. Therefore, the minimum number of inactivating mutations to increase the likelihood of tumor formation is one.

variable region of antibody

antigen binding region

B/B: Where in the human male reproductive system do the gametes become motile and capable of fertilization?
A) Testis
B) Urethra
C) Epididymis
D) Prostate gland

C) Epididymis The answer to this question is C because sperm, produced in the seminiferous tubules of the testes, completes maturation and becomes motile in the epididymis.

Testis

an organ which produces spermatozoa (male reproductive cells) and androgens, primarily testosterone

Urethra

duct by which urine is conveyed out of the body from the bladder, and which in male vertebrates also conveys semen

Epididymis

highly convoluted duct behind the testis, along which sperm passes to the vas deferens

Prostate gland

walnut sized gland located between the bladder and the penis secretes fluid that nourishes and protects sperm

B/B: According to the cross-bridge model of muscle contraction, the muscles stiffen after death because ATP is unavailable to bind and directly release:
A) ADP from the actin head.
B) ADP from the myosin head.
C) the actin head from the myosin filament.
D) the myosin head from the actin filament.

D) the myosin head from the actin filament. The answer to this question is D because during normal muscle contraction, ATP is required to break the bonds between the actin filament and the myosin head. After death, no new ATP is generated, so the myosin head cannot be released from the actin filament, resulting in stiffening of muscles.

Myosin-Actin Cross Bridge Cycle

B/B: A homodimeric protein was found to migrate through SDS polyacrylamide gel electrophoresis (SDS-PAGE) with a mobility that matched that of a 45-kDa standard. What change in the experiment would increase the chances of observing the mobility expected for the 22.5-kDa monomer?

A) Increasing the gel running time
B) Adding a reducing agent
C) Using a higher voltage
D) Removing the SDS

B) Adding a reducing agent The answer to this question is B because adding a reducing agent would eliminate any disulfide bridges and allow the monomers to run separately-thus leading to a migration expected for the 22.5-kDa protein.

SDS-PAGE

commonly used in the lab for the separation of proteins based on their molecular weight w/ reducing agent: denatures the protein by reducing disulfide linkages, overcoming some forms of tertiary protein folding

P/S: When comparing the color perception of humans and baboons, similarity in which anatomical structure of the eye is most important?
A) Optic disc
B) Sclera
C) Fovea
D) Lens

C) Fovea The answer to this question is C because the fovea is directly involved in color sensation and its distribution of receptors varies across different species.

P/S: If the researchers monitored the cortical activity of the baboons as they viewed the color patches, they would most likely find increased activity in the:
A) occipital cortex
B) temporal cortex
C) somatosensory cortex
D) motor cortex

A) occipital cortex The answer to the question is A because the occipital lobes are responsible for vision, including color vision.

P/S: Which schedule of reinforcement is used in the color-matching task?

"When a correct response was produced, the subject received a banana chip."

A) Fixed ratio
B) Variable ratio
C) Fixed interval
D) Variable interval

A) Fixed ratio The answer to this question is A. The passage states that the subjects received a banana chip every time they responded correctly, which is an example of a fixed ratio schedule of reinforcement.

P/S: The description of assimilation patterns in the passage (final paragraph) suggests that there is geographic variation in which pair of concepts?

"Variation regarding how immigrants adopt the local traditions and behavioral standards in a region can lead to distinct geographic patterns of assimilation."

A) Social status and roles
B) Social and cultural capital
C) Cultural values and norms
D) Material and symbolic culture

C) Cultural values and norms The answer to this question is C because cultural values and norms are identified in the passage’s reference to local traditions and behavioral standards. In the last paragraph of the passage, it is suggested that differences in the ways that immigrants adopt local traditions and behavioral standards can result in distinct geographic patterns of assimilation.

P/S: Which historical factor primarily accounts for the projected increase in the population aged 65 and older in the United States?
A) The increased immigration rate since the 1950s
B) The increased fertility rate after World War II
C) The sexual revolution of the 1960s and 1970s.
D) The relative deprivation of the Great Depression

B) The increased fertility rate after World War II The answer to this question is B. The increasing share of the population over the age of 65 primarily stems from the baby boomers, the post-World War II generation in the United States and Canada. Birth rates were relatively high for almost two decades after World War II. Demographers define the baby boom generation as those individuals born between approximately 1946 and 1964. The baby boom generation is the main sociohistorical factor that explains the projection in the passage about the increasing share of the population over 65 years of age.

P/S: To explain the demographic dynamics in the passage, a conflict theorist is most likely to consider the relationship among which factors?

A) Age, medicalization, and the illness experience
B) Generational status, social solidarity, and the welfare state
C) Age, social isolation, and networks of social support
D) Generational status, political power, and resource allocation

D) Generational status, political power, and resource allocation The answer to this question is D. The sociological paradigm of conflict theory broadly calls attention to competition among social groups, including generational conflict. Given the demographic dynamics raised in the passage, it is likely that competition over resources will emerge between the old and the young (for example, public support of health care versus education, each of which tends to benefit one age group more than the other). A conflict theorist would be interested in explaining how political power varies by generational status and thus affects the allocation of social resources.

P/S: The aging of the U.S. population is most likely to increase:
A) the dependency ratio.
B) the social gradient in health.
C) the life course perspective.
D) the intersectionality of medicine.

A) the dependency ratio. The answer to this question is A. The dependency ratio is a ratio of the number of economically dependent members of the population to the number of economically productive members. The economically dependent are those considered too young or too old to work, whereas the economically productive are the working-age population (approximately between the ages of 18 and 65).

P/S: Which finding best supports the linguistic relativity hypothesis?

A) All languages have a word for "up" and a word for "down".
B) Humans are better at learning words for primary colors than for secondary colors.
C) Some languages do no have words for "right" and "left".
D) Humans are better at distinguishing colors for which their language has a name.

D) Humans are better at distinguishing colors for which their language has a name. The answer to this question is D. The linguistic relativity hypothesis suggests that human cognition is affected by language.

P/S: A patient who is experiencing severe marital problems reports having no memory of any life events surrounding the marriage and the spouse. The patient’s memory for other life events is intact. This patient is most likely to be diagnosed with:
A) a conversion disorder
B) schizophrenia
C) retrograde amnesia
D) a dissociative disorder

D) a dissociative disorder The answer to this question is D. The patient is selectively forgetting distracting elements of his/her life, which indicates a dissociative disorder.

P/S: Some studies have found that increases in dopamine activity are associated with increased reward-seeking motivation. This finding suggests an association between dopamine levels and which type of learning?
A) Classical conditioning
B) Operant conditioning
C) Latent learning
D) Observational learning

B) Operant conditioning The answer to this question is B. The finding focuses on reward-seeking motivation, which is most closely associated with operant conditioning (change in behavior due to past outcomes).

P/S: Based on the passage, findings from comparative studies of social mobility have led some scholars to question which aspect of U.S. society?

"he proportion of the population with low incomes compared to the median income has been higher in the United States than in peer countries since at least 1980. Similarly, the United States tends to have less social mobility than peer countries. These findings from comparative research have led some scholars to question assumptions about opportunity in American society."

A) Meritocracy
B) Socialization
C) Social identity
D) Cultural capital

A) Meritocracy The answer to this question is A. According to the passage, the United States tends to have less social mobility than its peer countries. This empirical finding from comparative research had led some scholars to question assumptions about opportunity in American society. Such questioning of opportunity would be relevant to the concept of meritocracy, which assumes that opportunity is based on a combination of talent and effort.

P/S: Which type of poverty is referenced in the passage?

"The proportion of the population with low incomes compared to the median income has been higher in the United States than in peer countries since at least 1980."

A) Absolute poverty
B) Marginal poverty
C) Relative poverty
D) Structural poverty

C) Relative poverty The answer to this question is C. The section on social structural factors contains two references to the median income in the United States that both involve comparisons. Relative poverty refers to social disadvantage by income or wealth as compared to the social advantages linked to income or wealth in a society.

P/S: Which hypothesis of cultural assimilation, social support, and health outcomes is best supported by the passage information about immigrant groups? More assimilated groups will have:

A) lower levels of support and better overall health.
B) higher levels of support and worse overall health.
C) lower levels of support and worse overall health.
D) higher levels of support and better overall health.

C) lower levels of support and worse overall health. The answer to this question is C. In the section on social interaction factors, it is suggested that strong social support in local immigrant communities may partly explain the relatively good health of individuals from some immigrant groups in the United States (when compared to U.S.-born individuals with otherwise similar demographic characteristics). Immigrant groups that are more assimilated (or as they become assimilated) tend to have worse health outcomes (or lose their previous health advantages) than less assimilated immigrant groups. Thus, the correct response reflects the hypothesis that more assimilated groups are likely to have less social support over time.

P/S: Which information from the passage contradicts the socioeconomic gradient in health?

A) The U.S. has a comparatively high median household income.
B) The U.S. health disadvantage exists for all social classes.
C) The U.S. lags behind peer countries in health indicators.
D) The U.S. health disadvantage is explained by social capital.

B) The U.S. health disadvantage exists for all social classes. The answer to this question is B. The socioeconomic gradient in health refers to the graded relationship between social class and health, in which each "step" up on the hierarchy of social stratification tends to be associated with better health. Although all of the options are relevant to social class and stratification, only the correct response points to a section of the passage that directly addresses a health gradient. In stating that the U.S. health disadvantage exists across social classes, this assertion from the passage challenges the protective aspect of advantaged social class that is implied by the socioeconomic gradient in health.

P/S: Which hypothetical result from the online questionnaire would provide evidence of the fundamental attribution error?

A) Some participants rank personality factors above environmental factors.
B) Some participants rank genetic factors above personality factors.
C) Some participants rank environmental factors over both personality and genetic factors.
D) Some participants rank genetic factors over both personality and environmental factors.

A) Some participants rank personality factors above environmental factors. The answer to this question is A because the fundamental attribution error refers to stressing the importance of dispositional (i.e., personality) factors in one’s explanations of other people’s behavior and underemphasizing situational factors.

P/S: Which psychological concept best explains observed response times during the IAT?
A) Interference
B) Dissonance
C) Modeling
D) Schemas

D) Schemas The answer to this question is D. Schematic processing is relevant to explaining response times during the IAT because the speed with which memory schemas (organized clusters of knowledge) are activated and processed is presumed to indicate the participant’s implicit attitude.

P/S: Which component of attitudes is assessed by the study’s online questionnaire?
A) Affective
B) Behavioral
C) Cognitive
D) Subconscious

C) Cognitive The answer to this question is C. The questionnaire is described as assessing participants’ ideas about how people gain weight, as well as their beliefs about the relative contribution of genetics, personality, and environment in gaining, maintaining, and losing weight. This description identifies beliefs and ideas, which are part of the cognitive component of an attitude. The other possible answers are other components that are not relevant to the description of the questionnaire (which measured explicit attitudes).

P/S: Which concept best explains the comparison made in the passage regarding Internet responses, face-to-face interviews, and telephone surveys (paragraph 2)?
A) Selection bias
B) Attribution error
C) Social desirability
D) Stereotype threat

C) Social desirability The answer to this question is C. The last sentence of the second paragraph describes how research suggests that Internet responses may have advantages over face-to-face interviews or telephone surveys. For socially sensitive topics such as weight, participants may be more likely to provide socially acceptable answers during interviews or phone surveys. Internet responses do not elicit the same level of social desirability. The other possible answers are related psychological content, but do not explain the comparison made in the passage.

P/S: Which aspect of social interaction best explains the study’s findings related to BMI classification?

A) Primary versus secondary group ties
B) In-group versus out-group associations
C) Front-stage versus back-stage presentations
D) Role-taking versus impression management

B) In-group versus out-group associations The answer to this question is B because the findings related to BMI category suggest in-group versus out-group dynamics. The study found that the level of anti-obesity bias was related to the respondent’s BMI category. Bias tends to be directed toward out-group members; thus, it would be expected that weight-related bias would be associated with an individual’s BMI category.

P/S: Which research project provides the best application of the researchers’ suggestion for future study?

A) A survey of clinicians’ training in preventing medical conditions related to excess weight
B) A focus group that raises clinicians’ awareness of implicit bias and weight discrimination in health care
C) An experiment that measures changes in clinicians’ attitudes after interacting with patients who have different BMIs
D) An observational study of clinicians’ verbal and nonverbal communication with patients who have different BMIs

D) An observational study of clinicians’ verbal and nonverbal communication with patients who have different BMIs The answer to this question is D. The researchers’ suggestion calls for more studies that seek to understand how implicit attitudes about weight affect quality of care. This suggested research would address the relationship between healthcare providers’ attitudes and their behaviors. The correct answer provides a research method (observational study) that investigates communication behaviors (relevant to quality of care) with patients who differ based on BMI. Although the incorrect options could each provide valuable data, the focus remains primarily on providers’ attitudes. The correct response is consistent with the suggestion in the passage because it examines communication behaviors.

P/S: The provider attitudes study referenced at the end of the passage suggests that healthcare providers sometimes fail to consider which type of explanation of behavior?

"In a study of healthcare provider attitudes, researchers concluded that providers often overlooked the consequences of discrimination and poverty in explaining their patients’ behaviors."

A) Dispositional
B) Cognitive
C) Situational
D) Affective

C) Situational The answer to this question is C. The study cited in the question refers to the conclusion that attitudes of health providers often overlooked the consequences of discrimination and poverty in explaining their patients’ behavior. This statement suggests the need to consider situational explanations of patient behavior. The other possible answers are other types of psychological explanations, but none is suggested by the statement about provider attitudes.

P/S: The description of dependence and substance use disorders in the passage suggests which type of drug-related symptom?

"Although psychoactive drugs vary in terms of their risk of dependence, one of the factors associated with substance use disorders (SUD) in adolescents is the strong desire to ingest a drug (or other substance)"

A) Habituation
B) Tolerance
C) Withdrawal
D) Craving

D) Craving The answer to this question is D. Craving symptoms (strong desire to ingest a drug) are consistent with the description in the relevant passage section (identified in the question).

P/S: Which type of psychoactive drug has the lowest risk of dependence?
A) Stimulants
B) Hallucinogens
C) Alcohol
D) Sedatives

B) Hallucinogens The answer to this question is B. Hallucinogens have low risk of dependence, whereas the other substances listed as possible answers carry a moderate to high risk of physical or psychological dependence (the question does not require making a distinction between either).

P/S: A conflict theorist is most likely to reference which concept in order to explain the causes of access disparities for SUD treatment?
A) Stratification
B) Racialization
C) Socialization
D) Gentrification

A) Stratification The answer to this question is A because conflict theory is most often associated with class-based conceptions of society. Thus, social stratification is the concept that a conflict theorist would suggest is most relevant to explaining the access disparities from the passage.

P/S: A dopamine agonist is found to have dose-dependent effects. Based on this finding, which correlation is most likely to be supported?

A) A positive correlation between the dose of the dopamine agonist and the duration of the visual hallucinations that occur when the dopamine agonist is administered
B) A negative correlation between the dose of the dopamine agonist and the duration of the visual hallucinations that occur when the dopamine agonist is administered
C) A positive correlation between the dose of the dopamine agonist and the intensity of euphoria experienced when the dopamine agonist is administered
D) A negative correlation between the dose of the dopamine agonist and the intensity of euphoria experienced when the dopamine agonist is administered

C) A positive correlation between the dose of the dopamine agonist and the intensity of euphoria experienced when the dopamine agonist is administered The answer to this question is C. A dopamine agonist enhances the functioning of dopamine in the nervous system. Because the agonist is said to have dose-dependent effects, a positive correlation between the dose and the intensity of euphoria experienced is most likely.

P/S: A physician approaches a new patient with the assumption that the patient is not well educated and thus less knowledgeable about health issues. Does this scenario illustrate discrimination?

A) Yes; the scenario illustrates a judgment that is not based on supporting evidence.
B) No; the scenario identifies a bias directed at an individual rather than at a group.
C) Yes; the scenario suggests that a negative evaluation could affect the interaction.
D) No; the scenario describes an attitude but does not specify differential treatment.

D) No; the scenario describes an attitude but does not specify differential treatment. The answer to this question is D. The hypothetical physician displays a biased attitude, and thus a prejudice potentially based on a stereotype. However, no action or behavior is specifically identified with the scenario in the question. Without a description of differential treatment or behavior, discrimination is not identified.

P/S: A study finds that individuals who follow a weekly exercise routine exhibit less immunity decline compared to those who do not. The exercise group only included participants who exercised regularly, whereas the comparison group only included those who did not exercise regularly. Which statement does NOT identify a limitation of this research design?
A) The study lacks random assignment.
B) The study has a possible sampling bias.
C) The study lacks replicable results.
D) The study has a possible confound.

C) The study lacks replicable results. The answer to this question is C. The study lacks random assignment because the two groups were composed of individuals with established exercise habits. Because the participants were not equally likely to be in the two groups, there is a potential bias in the sample. In addition, exercise habits pose a potential confound because the group that already exercises regularly could differ from the comparison group in a number of ways. However, there is nothing about the hypothetical study that excludes the possibility of replicable results. Keeping in mind that the question asks for what is NOT a limitation, the incorrect options identify important methodological limitations of the study whereas the correct answer does not.

P/S: To determine the effectiveness of brainstorming, a researcher designs a study in which participants are asked to produce alternatives to an existing marketing strategy on their own or with a group. Which pattern of results is most likely based on research on group processes?
A) Groups arrive at the improved alternatives more often than individuals.
B) Groups are more likely to critically evaluate alternatives than individuals.
C) On average, participants generate more alternatives alone than in a group.
D) On average, participants produce more alternatives in a group than alone.

D) On average, participants produce more alternatives in a group than alone. The answer to this question is C. Social loafing refers to the fact that people are more productive alone than in a group. Research also suggests that individuals are less critical and less creative in groups.

Which one of Piaget’s cognitive developmental stages are all participants LEAST likely to have acquired? (10-13 y/o)
A) Concrete operational stage
B) Sensorimotor stage
C) Preoperational stage
D) Formal operational stage

D) Formal operational stage The answer to this question is D. Individuals 12 years and older reach Piaget’s formal operational stage of cognitive development, acquiring abstract reasoning skills. Participants in the study are ages 10-13 years. Thus, not all participants will already have acquired formal operations.

Compared to non-anxious participants, socially anxious participants reading negative peer evaluations are more likely to show enhanced activation in which brain region due to intense emotional responses?
A) Thalamus
B) Cerebellum
C) Amygdala
D) Medulla oblongata

C) Amygdala The answer to this question is C. The amygdala plays a role in intense emotional reactions.

P/S: Based on the results of studies 1 and 2, which conclusion about human perceptual development is supported?

A) Perceptual organization is innate and constant over time.
B) Different types of perceptual organization become functional over time.
C) Different types of perceptual organization cannot be reliably measured during infancy due to differences represented in the results.
D) Perceptual organization represented in the studies is mainly due to top-down processes.

B) Different types of perceptual organization become functional over time. The answer to this question is B. The results show that infant preference for a novel stimulus depends on the age of the infant. Younger infants show preference for lightness-darkness perception but not form perception, and older infants show a preference for form perception. This difference suggests that different types of perceptual organization mature over time.

P/S: Participants in the studies are in which stage of Erikson’s psychosocial development? (3-4 and 6-7 months)
A) Trust vs. mistrust
B) Competence vs. inferiority
C) Autonomy versus shame and doubt
D) Initiative vs. guilt

A) Trust vs. mistrust The answer to this question is A. Infants in the studies were 3 to 4 months old and 6 to 7 months old. According to Erikson, infants at this age are faced with issues of trust versus mistrust.

P/S: According to Freud, participants in studies 1 and 2 are in which stage of development? (3-4 and 6-7 months)
A) Phallic stage
B) Latency stage
C) Anal stage
D) Oral stage

D) Oral stage The answer to this question is D. Infants in the studies were 3 to 4 months old and 6 to 7 months old. According to Freud, the oral stage occurs from birth to 12-18 months and is characteristic of interest in oral gratification from sucking, eating, and biting.

P/S: Based on the passage, which statement is the best definition of groupthink? Groupthink refers to:

A) the faulty decisions that result when a powerful leader imposes his or her will on the group and overrides the wishes of the majority of the members.
B) a style of decision-making that occurs when strangers are placed into decision-making groups and forced to make choices under time pressure.
C) the presence of conformity pressures and beliefs of superiority that lead decision-making groups to fail to critically evaluate their alternatives and options.
D) a conscious decision-making strategy employed by groups tasked with making important policy decisions.

C) the presence of conformity pressures and beliefs of superiority that lead decision-making groups to fail to critically evaluate their alternatives and options. The answer to this question is C. Groupthink occurs when situational pressures hinder groups from critically evaluating relevant information. A powerful leader makes groupthink more likely, and the other group members are largely complicit in the behaviors and beliefs that produce groupthink. Groups affected by groupthink wrongly believe they have followed a sound decision-making process.

P/S: Based on the findings in Study 2, which strategy for decision-making groups would be most likely to prevent groupthink?

A) Make certain that all decision-making groups are made up of groups of friends.
B) Whenever possible, avoid the selection of a group leader through popular vote.
C) Encourage a group norm of critical evaluation and dissent in decision-making.
D) Require decision-making groups to verify that groupthink did not influence their decision.

C) Encourage a group norm of critical evaluation and dissent in decision-making. The answer to this question is C. Those in the social identity condition exhibited more groupthink symptoms than those in the interpersonal condition. They make more rationalizations, introduce fewer facts, and discuss risks less frequently. Pressures to reach consensus and stifle dissent are key to groupthink, and thus a group norm that encouraged dissent and critical evaluation would address the central problem of groupthink.

P/S: Although the study examines a specific bias in group decision-making, similar biases can influence individual decision-making. Which of the following individual-level effect is most similar to groupthink?
A) Self-serving bias
B) Confirmation bias
C) Hindsight bias
D) Response bias

B) Confirmation bias As with a group affected by groupthink, an individual’s confirmation bias causes the person to seek, and attend to, only information that confirms his or her existing point of view and to ignore disconfirming evidence.

P/S: The left cerebral hemisphere in humans is most often linked with which cognitive function?
A) Visuospatial skills
B) Music perception
C) Vocabulary skills
D) Emotion processing

C) Vocabulary skills The answer to this question is C. Vocabulary skills tend to be lateralized to the left hemisphere, whereas visuospatial skills, music perception, and emotion processing tend to be lateralized to the right hemisphere.

P/S: Research has shown that when individuals study material right before going to sleep, they perform better on a recall measure upon awakening compared to individuals who study and then watch a movie before going to sleep. Which memory construct provides the most likely explanation for these results?
A) Misinformation
B) Primacy
C) Spreading activation
D) Interference

D) Interference The answer to this question is D. When people study new material, any new information introduced between the initial learning (i.e., encoding) and retrieval, such as viewing a movie, will interfere with memory consolidation.

P/S: Individuals who have the ability to delay gratification in pursuit of long-term rewards are most likely to be categorized as having which type of intelligence?
A) Analytical
B) Creative
C) Interpersonal
D) Emotional

D) Emotional The answer to this question is D. Emotional intelligence refers to the ability to perceive, express, understand, and manage one’s emotions. Emotionally intelligent people are self-aware and can delay gratification in pursuit of long-term rewards, rather than being overtaken by immediate impulses. None of the other answer choices is related to delaying gratification.

P/S: Which procedure is most appropriate for assessing the dependent variable in this study?

A) Having the participants fill out a validated questionnaire that screens for anxiety disorders
B) Having a trained practitioner assess the participants for anxiety disorders
C) Monitoring how many trials it takes for participants to avoid a response that results in an electric shock when the electric shock is signaled with a tone
D) Monitoring how many trials it takes for participants to be conditioned to perform a response that results in the termination of an electric shock

D) Monitoring how many trials it takes for participants to be conditioned to perform a response that results in the termination of an electric shock The answer to this question is D. The purpose of the study was to determine how anxiety disorders affect escape learning, and this option is the only one that refers to an empirical observation of escape behavior. A researcher is interested in how anxiety disorders affect escape learning.

P/S: Compared to control mice, the genetically modified mice were more likely to exhibit which expression levels of Aβ and NFT?

A) Higher expression levels of both Aβ and NFT in the brain regions examined in the study
B) Lower expression levels of both Aβ and NFT in the brain regions examined in the study
C) Lower expression levels of Aβ, but higher expression levels of NFT in the brain regions examined in the study
D) Higher expression levels of Aβ, but lower expression levels of NFT in the brain regions examined in the study

A) Higher expression levels of both Aβ and NFT in the brain regions examined in the study The answer to this question is A because AD is associated with a build-up of Aβ and NFT levels in certain brain regions. Therefore, the researchers would most likely overexpress both Aβ and NFT in the genetically modified mice to create a good mouse model for AD.

P/S: The conditioned stimulus in the study is:
A) encenicline
B) foot shock
C) the light
D) freezing behavior

C) the light The answer to this question is C. The conditioned stimulus is a stimulus that, only after being paired several times with an unconditioned stimulus (in this case, foot shock), triggers a conditioned response (freezing behavior).

P/S: The first phase of the fear conditioning paradigm, as described in the study, is known as:
A) discrimination
B) acquisition
C) shaping
D) generalization

B) acquisition The answer to this question is B. In the acquisition phase of the fear conditioning paradigm, a stimulus (light) that is neutral with respect to the freezing response is associated with an unconditioned stimulus (in this case, foot shock) over a series of trials, until the neutral stimulus elicits a conditioned response (freezing behavior in response to the light).

P/S: An independent variable in the study is: (X-axis: saline vs. encenicline treatment; Y-axis: percentage of time spent freezing)

A) Aβ versus NFT proteins.
B) spatial memory versus no spatial memory.
C) saline versus encenicline.
D) fear conditioning versus no fear conditioning.

C) saline versus encenicline. The answer to this question is C. Subjects were administered either saline or the acetylcholine agonist, encenicline. Thus, these substances were the two levels of the independent variable.

P/S: Which observation is LEAST likely to be made in an examination of the brains of the genetically modified mice? (modeling Alzheimer’s disease)

A) Increased synaptic connections between neurons
B) Malfunction of transporting nutrients along axons
C) Extracellular aggregation of neurofibrillary tangles
D) Increased expression of apoptotic markers

A) Increased synaptic connections between neurons The answer to this question is A because the presence of Aβ plaques and neurofibrillary tangles will not allow for increased connection between neurons in the brains of the genetically modified mice. The other responses represent declines in cellular function and survival that are more likely to be associated with AD.

P/S: Which statement accurately represents the application of CBT in the study? In the study, CBT addresses smoking behaviors through:
A) examining personal histories for the causal mechanism behind behavior
B) associating negative states with unwanted behavior
C) observation of others in order to model effective and healthy behaviors
D) systematic modification of individual behavior and self-assessments

D) systematic modification of individual behavior and self-assessments The answer to this question is D. CBT first addresses maladaptive behaviors through behavior therapy to systematically modify a person’s behavior. This is followed by sessions designed to foster cognitive change, through self-assessments.

P/S: How would a structural functionalist interpret the efficacy of yoga as part of a smoking cessation therapy?

A) Yoga provides an alternate understanding of healthful practices that enables the individual to better understand his or her personal needs and motives.
B) Yoga provides an inexpensive therapy option for those lacking the financial resources necessary for more expensive medical interventions.
C) The utility of yoga as an effective smoking cessation therapy stems from the transformation of the individual’s self-concept as a nonsmoker.
D) The utility of yoga as an effective smoking cessation therapy is an unintended, though beneficial, outcome of a yoga practice.

D) The utility of yoga as an effective smoking cessation therapy is an unintended, though beneficial, outcome of a yoga practice. The answer to this question is D because it describes a latent, or unintended, function. Because the expected function of yoga is not specifically smoking cessation, its utility as a cessation therapy is a latent function of the social activity. The sociological paradigm of functionalism makes a distinction between manifest, or intended, and latent, or unintended, functions of social activities. From the functionalist perspective, almost all social actions have both manifest functions and latent functions, both of which are connected to overall social stability. The other response options with the question are better linked to the conflict or symbolic interactionist perspectives in sociology.

P/S: Which statement reflects the medicalization of smoking and smoking cessation?

A) Smoking is a personal behavior that can be adjusted through individual behavior modification.
B) Smoking is an unhealthy behavior that can be structurally managed via legislation to reduce overall smoking rates.
C) Smoking is an addictive behavior that can be treated via the use of pharmaceutical intervention.
D) Smoking is a public health issue that is socially desirable to mitigate through smoking cessation programs.

C) Smoking is an addictive behavior that can be treated via the use of pharmaceutical intervention. The answer to this question is C because it suggests the expansion of medical authority to smoking cessation and identifies a specific medical therapy. Medicalization refers to the taken-for-granted process in which a problem comes to be defined and treated by the social institution of medicine. A behavior undergoes medicalization when both the definition of the problem and the therapy intended to improve it are couched in medical terms. The other options do not suggest the expansion of medical authority or do not describe both problem and solution in medical terms.

Results from a study indicate that there is a -0.82 correlation between alcohol consumption and internal locus of control. Which statement will most likely be concluded from these findings?
A) High alcohol consumption behavior is strongly associated with people perceiving that they control their own fate.
B) High alcohol consumption behavior is strongly associated with people perceiving that chance or outside forces control their fate.
C) High alcohol consumption behavior causes people to perceive that chance or outside forces determine their fate.
D) High alcohol consumption behavior is weakly associated with people perceiving that they control their own fate.

B) High alcohol consumption behavior is strongly associated with people perceiving that chance or outside forces control their fate. The answer to this question is B. The strong negative correlation means that, as one variable increases (in this case, people who drink more alcohol), there is an associated decrease in another variable (in this case, internal locus of control, or believing that they control their own fate).

P/S: A researcher conducts observational research on the study habits of college students. When students are aware of the researcher’s presence, they are more attentive, focused, and structured. When students are not aware of the researcher’s presence, they are inattentive, unfocused, and distracted. Which concept best describes this phenomenon?

A) Impression management
B) Hawthorne effect
C) Self-fulfilling prophecy
D) Thomas theorem

B) Hawthorne effect The answer to this question is B. The Hawthorne effect describes changes in research participants as a result of their awareness that they are being observed. The changes to the students’ study habits are best explained by the Hawthorne effect, which was first observed among workers. The incorrect options refer to related but distinct concepts. A self-fulfilling prophecy is an individual’s internalization of a label that leads to a fulfillment of that label. Impression management refers to individuals actively managing how they are perceived by others. The Thomas theorem states that if an individual believes something to be real, then it is real in its consequences.

impression management

refers to individuals actively managing how they are perceived by others

Hawthorne effect

changes in research participants as a result of their awareness that they are being observed

self-fulfilling prophecy

an individual’s internalization of a label that leads to fulfillment of that label

Thomas theorem

states that if an individual believes something to be real, then it is real in its consequences

If an individual loses his job and obtains a similar amount of income through social security disability insurance. Which type of social mobility best characterizes the situation?
A) Exchange mobility
B) Horizontal mobility
C) Downward mobility
D) Upward mobility

B) Horizontal mobility The answer to this question is B. The most important information in the question is the reference to a similar amount of income. Horizontal mobility describes a situation in which an individual changes some aspect of social identity (from employed to unemployed, for example) but maintains the same relative status (income remains the same).

Share This
Flashcard

More flashcards like this

NCLEX 10000 Integumentary Disorders

When assessing a client with partial-thickness burns over 60% of the body, which finding should the nurse report immediately? a) ...

Read more

NCLEX 300-NEURO

A client with amyotrophic lateral sclerosis (ALS) tells the nurse, "Sometimes I feel so frustrated. I can’t do anything without ...

Read more

NASM Flashcards

Which of the following is the process of getting oxygen from the environment to the tissues of the body? Diffusion ...

Read more

Unfinished tasks keep piling up?

Let us complete them for you. Quickly and professionally.

Check Price

Successful message
sending